[obm-l] Re: [obm-l] Re: Demosntração desigualdade

2016-09-08 Por tôpico Esdras Muniz
estions/1917400/inequality-o >> n-six-variables >> >> O cara deu um contra exemplo que a desigualdade é falsa, mas não vejo >> nenhum errro na minha demonstração, alguém poderia me dizer qual é o erro? >> > > > -- > Esta mensagem foi verificada pelo sist

RE: [obm-l] Ajuda em Aritmética

2016-09-26 Por tôpico Esdras Muniz
Se p é um primo diferente de 5, os restos dos outros 2 por 5 são os mesmos que os de p^2-1 e p^2+1 respectivamente. Se os 3 números são primos, nenhum deles é múltiplo de 5. Daí o produto (p^2-1)(p^2+1) não pode ser múltiplo de 5. Mas esse produto é p^4-1. Mas o pequeno teorema de Fermat garante

[obm-l] Re: [obm-l] Teoria dos Números

2016-09-26 Por tôpico Esdras Muniz
ncia, mas eu travei nessa questão. > > Por favor, algum colega poderia fazer a demonstração? > > -- > Esta mensagem foi verificada pelo sistema de antivírus e > acredita-se estar livre de perigo. -- Esdras Muniz Mota Mestrando em Matemática Universidade Federal do Ceará --

[obm-l] Re: [obm-l] aritmética

2016-10-12 Por tôpico Esdras Muniz
2^99 + 14^99 por 169 > > -- > Esta mensagem foi verificada pelo sistema de antivírus e > acredita-se estar livre de perigo. > -- Esdras Muniz Mota Mestrando em Matemática Universidade Federal do Ceará -- Esta mensagem foi verificada pelo sistema de antiv�rus e acredita-se estar livre de perigo.

[obm-l] Recorrência

2016-10-16 Por tôpico Esdras Muniz
Olá amigos, gostaria que me passassem eferências de livros ou artigos que falem sobre recorrência. Dês de já obrigado. -- Esdras Muniz Mota Mestrando em Matemática Universidade Federal do Ceará -- Esta mensagem foi verificada pelo sistema de antiv�rus e acredita-se estar livre de perigo.

[obm-l] Re: [obm-l] Re: [obm-l] Re: [obm-l] Recorrência

2016-10-16 Por tôpico Esdras Muniz
ciples and Techniques in Combinatorics >> ( Chen chuan-chong ) acredito ser intermediário pra Phoda >> Aí desses pesados existe o Introduction to Combinatorics e o >> Problems in Combinatorics and Graph Theory ambos do renomado IOAN TOMESCU >> >> Em domingo, 16 de outu

RE: [obm-l] Divisibilidade Simultânea

2016-10-17 Por tôpico Esdras Muniz
Sim, m = n =1. -Mensagem Original- De: "Richard Vilhena" Enviada em: ‎17/‎10/‎2016 20:41 Para: "obm-l@mat.puc-rio.br" Assunto: [obm-l] Divisibilidade Simultânea Gostaria que uma ajuda. Obrigado! É possível encontrar inteiros m > 0, n > 0, tal que (n + 1)|(m2 + 1) e simultaneamente (m

Re: [obm-l] Problema de geometria.

2016-11-02 Por tôpico Esdras Muniz
a > > -- > Esta mensagem foi verificada pelo sistema de antivírus e > acredita-se estar livre de perigo. -- Esdras Muniz Mota Mestrando em Matemática Universidade Federal do Ceará -- Esta mensagem foi verificada pelo sistema de antiv�rus e acredita-se estar livre de perigo.

[obm-l] Re: [obm-l] Re: [obm-l] Re: [obm-l] Re: [obm-l] Re: [obm-l] Re: [obm-l] Re: [obm-l] Re: [obm-l] Polinômio irredutível em Z

2016-11-24 Por tôpico Esdras Muniz
>>> >>>>>>> >>>>>>> >>>>>>> = >>>>>>> Instru?es para entrar na lista, sair da lista e usar a lista em >>>>>>> http://www.mat.puc-rio.br/~obmlistas/obm-l.html >>>>>>> >>>>>>> = >>>>>>> >>>>>>> >>>>>>> >>>>>> >>>>>> >>>>>> >>>>>> This message was sent using IMP, the Internet Messaging Program. >>>>>> >>>>>> >>>>>> >>>>>> -- >>>>>> Esta mensagem foi verificada pelo sistema de antivírus e >>>>>> acredita-se estar livre de perigo. >>>>>> >>>>>> >>>>>> >>>>>> = >>>>>> Instruções para entrar na lista, sair da lista e usar a lista em >>>>>> http://www.mat.puc-rio.br/~obmlistas/obm-l.html >>>>>> >>>>>> = >>>>>> >>>>> >>>>> >>>>> -- >>>>> Esta mensagem foi verificada pelo sistema de antivírus e >>>>> acredita-se estar livre de perigo. >>>> >>>> >>>> -- >>>> Esta mensagem foi verificada pelo sistema de antivírus e >>>> acredita-se estar livre de perigo. >>>> >>> >>> >>> -- >>> Esta mensagem foi verificada pelo sistema de antivírus e >>> acredita-se estar livre de perigo. >>> >> >> >> -- >> Esta mensagem foi verificada pelo sistema de antivírus e >> acredita-se estar livre de perigo. >> > > > -- > Esta mensagem foi verificada pelo sistema de antivírus e > acredita-se estar livre de perigo. > -- Esdras Muniz Mota Mestrando em Matemática Universidade Federal do Ceará -- Esta mensagem foi verificada pelo sistema de antiv�rus e acredita-se estar livre de perigo.

[obm-l] Re: [obm-l] Qual a maior potência?

2017-01-16 Por tôpico Esdras Muniz
aior potência? 4^53 ou 5^44. > > -- > Esta mensagem foi verificada pelo sistema de antivírus e > acredita-se estar livre de perigo. -- Esdras Muniz Mota Mestrando em Matemática Universidade Federal do Ceará -- Esta mensagem foi verificada pelo sistema de antiv�rus e acredita-se estar livre de perigo.

Re: [obm-l] desigualdade

2017-05-07 Por tôpico Esdras Muniz
mensagem foi verificada pelo sistema de antivírus e > >> acredita-se estar livre de perigo. > >> > >> > >> -- > >> Esta mensagem foi verificada pelo sistema de antivírus e > >> acredita-se estar livre de perigo. > > > > > > > > -- > > Esta mensagem foi verificada pelo sistema de antivírus e > > acredita-se estar livre de perigo. > > -- > Esta mensagem foi verificada pelo sistema de antivírus e > acredita-se estar livre de perigo. > > > = > Instru�ões para entrar na lista, sair da lista e usar a lista em > http://www.mat.puc-rio.br/~obmlistas/obm-l.html > = > -- Esdras Muniz Mota Mestrando em Matemática Universidade Federal do Ceará -- Esta mensagem foi verificada pelo sistema de antiv�rus e acredita-se estar livre de perigo.

Re: [obm-l] boatos sobre elon lages lima

2017-05-23 Por tôpico Esdras Muniz
gt; Esta mensagem foi verificada pelo sistema de antivírus e > acredita-se estar livre de perigo. -- Esdras Muniz Mota Mestrando em Matemática Universidade Federal do Ceará -- Esta mensagem foi verificada pelo sistema de antiv�rus e acredita-se estar livre de perigo.

Re: [obm-l] Desigualdade

2017-05-27 Por tôpico Esdras Muniz
Solução muito boa. Em 27 de mai de 2017 00:37, "Gabriel Tostes" escreveu: > Tira ln, esse produto vai ser: > Sum{n>=1} ln(n+1)/(2^n) = M > > Bora escrever M de outro jeito: > > M= ln(2) + [ln(3)-ln(2)]/2 + [ln(4)-ln(3)]/2^2 + ... > > M= Sum{n>=1} (ln(n+1)-ln(n))/2^(n-1) > > Como ln(n+1)-ln(n)=ln

Re: [obm-l] Desigualdade

2017-05-28 Por tôpico Esdras Muniz
tá provado. Peço a ajuda de vocÊs para provar o lema. Lema: Considere a sequência a_n=(a_(n-1)^2)/n, onde a_1=3 então (a_n)^2>n+1. Douglas Oliveira Em 27 de maio de 2017 18:10, Esdras Muniz escreveu: > Solução muito boa. > > Em 27 de mai de 2017 00:37, "Gabriel Tostes&quo

Re: [obm-l] Radicais

2017-06-04 Por tôpico Esdras Muniz
rificada pelo sistema de antivírus e >> acredita-se estar livre de perigo. > > > -- > Esta mensagem foi verificada pelo sistema de antivírus e > acredita-se estar livre de perigo. > -- Esdras Muniz Mota Mestrando em Matemática Universidade Federal do Ceará -- Esta mensagem foi verificada pelo sistema de antiv�rus e acredita-se estar livre de perigo.

[obm-l] Re: [obm-l] Re: [obm-l] Re: [obm-l] Sugestão de material para OBM

2017-07-04 Por tôpico Esdras Muniz
>> -- >>> Esta mensagem foi verificada pelo sistema de antivírus e >>> acredita-se estar livre de perigo. >> >> >> >> -- >> Esta mensagem foi verificada pelo sistema de antivírus e >> acredita-se estar livre de perigo. >> > > > -- > Esta mensagem foi verificada pelo sistema de antivírus e > acredita-se estar livre de perigo. > -- Esdras Muniz Mota Mestrando em Matemática Universidade Federal do Ceará -- Esta mensagem foi verificada pelo sistema de antiv�rus e acredita-se estar livre de perigo.

Re: [obm-l] a quem possa interessar

2017-08-10 Por tôpico Esdras Muniz
>> https://drive.google.com/drive/folders/0B8qeUE5SqcPAWFVaM1N5anN3S2M >> >> >> >> -- >> Esta mensagem foi verificada pelo sistema de antivírus e >> acredita-se estar livre de perigo. > > > > -- > Esta mensagem foi verificada pelo sistema de anti

Re: [obm-l] Fibonacci teoria dos numeros

2017-08-31 Por tôpico Esdras Muniz
; Douglas Oliveira. > > -- > Esta mensagem foi verificada pelo sistema de antivírus e > acredita-se estar livre de perigo. -- Esdras Muniz Mota Mestrando em Matemática Universidade Federal do Ceará -- Esta mensagem foi verificada pelo sistema de antiv�rus e acredita-se estar livre de perigo.

Re: [obm-l] Problema de grafos

2017-09-02 Por tôpico Esdras Muniz
Cada vértice pode ter como grau um número de 0 a n-1, porém o 0 e o n-1 não podem ambos ser graus de vértices, pois se um tem grau n-1 então ele está ligado a todos os outros vértices. Então há apenas n-1 possibilidades para o grau de cada vértice. Pelo pcp há dois vértices com o mesmo grau. Em 2

[obm-l] Re: [obm-l] Dúvida combinatória

2017-11-14 Por tôpico Esdras Muniz
; Eduardo > > -- > Esta mensagem foi verificada pelo sistema de antivírus e > acredita-se estar livre de perigo. > -- Esdras Muniz Mota Mestrando em Matemática Universidade Federal do Ceará -- Esta mensagem foi verificada pelo sistema de antiv�rus e acredita-se estar livre de perigo.

[obm-l] Re: [obm-l] Dúvida combinatória

2017-11-14 Por tôpico Esdras Muniz
Obs: $$S(n,\,k_1,\cdots ,k_n)=\frac{n!}{(k_1!\cdots k_n!)(1!)^{k_1}\cdots (n!)^{k_n}}$$ -- Esta mensagem foi verificada pelo sistema de antiv�rus e acredita-se estar livre de perigo.

Re: [obm-l] Combinatoria - quantas sequencias de comprimento "n" , com "p" elementos

2018-02-27 Por tôpico Esdras Muniz
p^n-(p-1)^n Em 27 de fev de 2018 09:57, "Claudio Buffara" escreveu: > Isso é igual ao número de sobrejeções de um conjunto com n elementos num > conjunto com p elementos. > > É igual a p!*S(n,p), onde S(n,p) é o número de Stirling do 2o tipo (= > número de partições de um conjunto com n element

[obm-l] Re: [obm-l] Re: [obm-l] Re: [obm-l] Fwd: não sei como fazer, tentei desigualdades de médias e não saiu

2018-05-13 Por tôpico Esdras Muniz
pelo sistema de antivrus e >> acredita-se estar livre de perigo. >> >> >> >> -- >> Esta mensagem foi verificada pelo sistema de antivírus e >> acredita-se estar livre de perigo. >> > -- > Fiscal: Daniel Quevedo > > -- > Esta mensagem foi verificada pelo sistema de antivírus e > acredita-se estar livre de perigo. > -- Esdras Muniz Mota Mestrando em Matemática Universidade Federal do Ceará -- Esta mensagem foi verificada pelo sistema de antiv�rus e acredita-se estar livre de perigo.

[obm-l] Re: [obm-l] área de triângulo( compartilhando)

2018-05-13 Por tôpico Esdras Muniz
< marconeborge...@hotmail.com> escreveu: > As medidas de dois lados de um triângulo são a e b e sua área é igual a > (a^2+b^2)/4 > > Determine os ângulos do triângulo > > -- > Esta mensagem foi verificada pelo sistema de antivírus e > acredita-se estar livre de peri

Re: [obm-l] geometria plana

2018-07-29 Por tôpico Esdras Muniz
- >> Esta mensagem foi verificada pelo sistema de antivírus e >> acredita-se estar livre de perigo. >> > > > -- > Esta mensagem foi verificada pelo sistema de antivírus e > acredita-se estar livre de perigo. > > > -- > Esta mensagem foi verificada pelo s

[obm-l] Re: [obm-l] Polinômios ( RPM)

2018-09-20 Por tôpico Esdras Muniz
equar para > aplica-las . > > -- > Esta mensagem foi verificada pelo sistema de antivírus e > acredita-se estar livre de perigo. -- Esdras Muniz Mota Mestrando em Matemática Universidade Federal do Ceará -- Esta mensagem foi verificada pelo sistema de antiv�rus e acredita-se estar livre de perigo.

[obm-l] Re: [obm-l] trigonometria, alguma sugestão?

2018-09-28 Por tôpico Esdras Muniz
a igualdade x + y + z = 2jπ/k é verdadeira. > > -- > Israel Meireles Chrisostomo > > -- > Esta mensagem foi verificada pelo sistema de antivírus e > acredita-se estar livre de perigo. -- Esdras Muniz Mota Mestrando em Matemática Universidade Federal do Ceará -- Esta mensage

[obm-l] Re: [obm-l] Re: [obm-l] Recorrência de 2ª Ordem

2019-02-15 Por tôpico Esdras Muniz
Suponha que a equação seja Xn+2=2aXn+1-a^2Xn, então, (Xn+2-aXn+1)=a(Xn+1-aXn). Defina Yn=Xn+1-aXn. Daí, Yn+1=aYn, então fica Yn=B.a^n. Xn+1=aXn+B.a^n. Que é uma equação de primeira ordem. Em sex, 15 de fev de 2019 00:11, Claudio Buffara Pelo método experimental. > > Suponhamos que você já conheça

[obm-l] Re: [obm-l] Torneio das Cidades ( Número mínimo de Tentativas )

2019-02-24 Por tôpico Esdras Muniz
Já vi um problema parecido da OBM que dava pra resolver usando o teorema de Turan. Pra esse imaginei assim, VC separa as baterias em dois grupos, B das boas e R das ruins e liga duas se foram testadas juntas. O numero máximo de arestas que dá pra colocar sem a lampada acender é |B|×|R| +|R|×(|R|-1)

Re: [obm-l] desigualdades

2019-06-12 Por tôpico Esdras Muniz
Provar que E=a/(a+b) + b/(b+c) + c/(c+d) + d/(d+a) < 3 Se u, v e k são positivos, com uhttps://www.avast.com/sig-email?utm_medium=email&utm_source=link&utm_campaign=sig-email&utm_content=webmail> Livre de vírus. www.avast.com

[obm-l] Re: [obm-l] Questão sobre equações funcionais

2019-07-28 Por tôpico Esdras Muniz
Mas essa função que VC achou não satisfaz a igualdade. Em dom, 28 de jul de 2019 01:05, Carlos Monteiro < cacacarlosalberto1...@gmail.com> escreveu: > (Questão) Encontre todas as funções f : R-> R tais que > f(xy - f(x)) = x.f(y) > > Minha tentativa, não sei se está correta: > I) p(x, f(x)/(x-1)

[obm-l] Re: [obm-l] Questão sobre equações funcionais

2019-07-28 Por tôpico Esdras Muniz
Errei, satisfaz sim :) Em dom, 28 de jul de 2019 14:21, Esdras Muniz escreveu: > Mas essa função que VC achou não satisfaz a igualdade. > > Em dom, 28 de jul de 2019 01:05, Carlos Monteiro < > cacacarlosalberto1...@gmail.com> escreveu: > >> (Questão) Encontre todas as

[obm-l] Re: [obm-l] Re: [obm-l] Congruência (?)

2019-10-04 Por tôpico Esdras Muniz
>> Saudações, >> PJMS >> >> >> >> >> Em qui, 3 de out de 2019 às 17:51, marcone augusto araújo borges < >> marconeborge...@hotmail.com> escreveu: >> >>> Se n é um número natural par não divisível por 10, quais são os dois >>> últimos algarismos de n^20? >>> -- >>> Esta mensagem foi verificada pelo sistema de antivírus e >>> acredita-se estar livre de perigo. >>> >> > -- > Esta mensagem foi verificada pelo sistema de antivírus e > acredita-se estar livre de perigo. -- Esdras Muniz Mota Mestrando em Matemática Universidade Federal do Ceará -- Esta mensagem foi verificada pelo sistema de antiv�rus e acredita-se estar livre de perigo.

[obm-l] Re: [obm-l] Teorema Fundamental da álgebra prova

2019-10-10 Por tôpico Esdras Muniz
Se o polinômios tem grau ímpar, vc consegue mostrar que ele tem uma raíz real, usando só a continuidade do polinômio. Tem tb uma demonstração elementar de um caso particular do tfa, o caso em que n/4 e 3n/4 não são quadrados perfeitos, onde n é o grau do polinômio. Em qui, 10 de out de 2019 16:12,

Re: [obm-l] Cotangentes

2019-10-23 Por tôpico Esdras Muniz
Isso é falso, pois (2k-1)/4n forma uma seq crescente indo pro infinito, então a cota gente ao quadrado forma uma seq decrescente indo pra zero. Em qua, 23 de out de 2019 16:52, Israel Meireles Chrisostomo < israelmchrisost...@gmail.com> escreveu: > Como posso mostrar que cot²((2k-1)/4n) com k=1 a

Re: [obm-l] Polinomios

2019-10-27 Por tôpico Esdras Muniz
Dá pra provar por indicação, suponha q o resultado vale pra grau de P<=n-1. Daí, use que entre um máximo e um mínimo de P, há no máximo uma raíz (é fácil mostrar isso usando só a continuidade de P). Assim, por suposição, P tem no máximo n+1 máximos, que são as raízes de P', + infinito e - infinito.

[obm-l] Re: [obm-l] Triângulos.

2019-10-27 Por tôpico Esdras Muniz
Usa ma>=mg Em dom, 27 de out de 2019 19:27, Guilherme Abbehusen < gui.abbehuse...@gmail.com> escreveu: > Olá, poderiam me ajudar com essa questão? > > A hipotenusa de um triângulo retângulo tem medida igual "a" e os catetos > medidas iguais a "b" e "c" . Qual é o valor mínimo da equação: a/(b*c)^

[obm-l] Re: [obm-l] Séries e somatórios

2019-10-30 Por tôpico Esdras Muniz
O livro concrete mathematics fala disso. Em qua, 30 de out de 2019 19:51, Alexandre Antunes < prof.alexandreantu...@gmail.com> escreveu: > > Boa noite, > > Alguém tem alguma referência de livro/apostila sobre operações e > propriedades "avançadas" sobre séries, somatórios, somatórios duplos, etc.

[obm-l] Re: [obm-l] Máximo e Mínimo de uma Função

2019-11-02 Por tôpico Esdras Muniz
O máximo e o mínimo dessa função dependem do domínio onde ela está definida, por exemplo, se ela está definida em R-{0}, ela não tem máximo nem mínimo. Isso interpretando que a questão quer literalmente o valor máximo de f. Se interpretar que ela quer o valor de x para o qual f(x) é máximo ou mínim

Re: [obm-l] Bibliografia

2019-11-02 Por tôpico Esdras Muniz
Tem o "Elon fino" Análise real do Elon Lages Lima. Sobre limite, tem muita biografia em outras línguas, tem um livro muito bom em italiano, mas não sei se tem em inglês, o livro do Pagani. Em sex, 1 de nov de 2019 17:01, Luiz Antonio Rodrigues < rodrigue...@gmail.com> escreveu: > Olá, pessoal! >

[obm-l] Re: [obm-l] Re: [obm-l] Re: [obm-l] Minha solução para o item c) do problema 3 da prova da OBM-2017

2019-11-08 Por tôpico Esdras Muniz
Acho que é só passar 2017 para a base 6 e depois substituir os algarismos 0, 1, 2, 3, 4, 5 por 1, 3, 5, 7, 9 respectivamente. Assim, 2017 na base 6 é 13201, trocando os algarismos, fica: 37513. Em qui, 7 de nov de 2019 22:16, Cauã DSR escreveu: > Muito obrigado! É realmente uma honra ler isso.

Re: [obm-l]

2019-11-08 Por tôpico Esdras Muniz
Usa que f(x^2+x-3)=(x^3+2x^2-3x-5)f(x). Em qui, 7 de nov de 2019 11:53, gilberto azevedo escreveu: > [Polinômios] > > Dada a função f(x) = x³ + x² - 4x + 1 , mostrar que se f(r) = 0 , então > f(r² + r -3) = 0. > Creio que tem uma sacada pra aparecer r³ + r² - 4r + 1 , e usar que isso é > 0, poré

Re: [obm-l]

2019-11-12 Por tôpico Esdras Muniz
Dá para mostrar que a única solução com a e b pares é (2, 2). Agora com a e b ímpares, não consegui. Em ter, 12 de nov de 2019 18:19, Pedro José escreveu: > Boa noite! > Agora captei vosso pensamento. > Só que ao transformar a equação em uma equação de Pell, nós maculamos a > função 3^n. > Em ve

Re: [obm-l]

2019-11-12 Por tôpico Esdras Muniz
gt; grato pela luz, estava tão obsecado e só rodando em círculos, tal qual >>> patrulha perdida. >>> >>> Saudações, >>> PJMS >>> >>> Em ter., 12 de nov. de 2019 às 19:19, Esdras Muniz < >>> esdrasmunizm...@gmail.com>

[obm-l] Re: [obm-l] Re: [obm-l] CONTAGEM, SISTEMA DE NUMERAÇÂO E ORDENAÇÂO DE CONJUNTOS

2019-11-13 Por tôpico Esdras Muniz
Tá virando moda esse tipo de problema, já são ao menos 3 parecidos que o povo coloca aqui. Tem algum artigo ou livro pra estudar esse tipo de problema? Em qua, 13 de nov de 2019 16:24, Jamil Silva escreveu: > Só esqueci de dizer que as sequencias são impressas seguindo rigorosamente > a ordem al

[obm-l] Re: [obm-l] Re: [obm-l] Qual o 2020º termo da sequência abaixo ?

2019-11-17 Por tôpico Esdras Muniz
17 Em dom, 17 de nov de 2019 20:59, Jamil Silva escreveu: > Por que mod40 ? > > 17.11.2019, 14:36, "Claudio Buffara" : > > Me parece que basta calcular o 2020o termo sem a restrição de ser mod 40 > (é uma sequência de Fibonacci começando por 5 e 2) e depois ver quanto e’ > a(2020) mod 40, sendo

[obm-l] Re: [obm-l] Re: [obm-l] Qual o 2020º termo da sequência abaixo ?

2019-11-18 Por tôpico Esdras Muniz
Eu resolvi fazendo um programa, e deu 17. Mas a ideia é essa mesmo do mod 41. Se aparecerem dois números seguidos que já apareceram antes, a sequência começar a se repetir, tipo 1, 2,..., 1, 2,... E isso com certeza vai ocorrer, pois só há 41×40 duplas de números seguidos possíveis, considerando a

Re: [obm-l]

2019-11-22 Por tôpico Esdras Muniz
Eu usei mg>= mh Em sex, 22 de nov de 2019 17:04, Claudio Buffara escreveu: > Que podemos elevar ao quadrado, obtendo x^6/(x - 12). > > Ou seja, o problema se torna achar o valor mínimo de x^6/(x - 12), com x > > 12 (não pode ser "=" ...). > Depois, é só tirar a raiz quadrada. > > Agora, usamos

[obm-l] Re: [obm-l] Triângulos.

2019-11-22 Por tôpico Esdras Muniz
Acho que a questão pressupõe que os lados devem ser inteiros. Daí se os lados são x, y e z, com x<=yx^2+y^2 e z escreveu: > Do jeito que está escrito, uma infinidade. > > Enviado do meu iPhone > > > Em 22 de nov de 2019, à(s) 19:18, Guilherme Abbehusen < > gui.abbehuse...@gmail.com> escreveu: > >

[obm-l] Re: [obm-l] Re: [obm-l] Re: [obm-l] Triângulos.

2019-11-24 Por tôpico Esdras Muniz
Verdade, não tinha percebido. Em dom, 24 de nov de 2019 14:17, Pedro José escreveu: > Boa tarde! > Esdras, > Não seria z>=3. > 3, 2, 2 dá um obtusângulo. > > Saudações, > PJMS > > Em sáb, 23 de nov de 2019 01:52, Esdras Muniz > escreveu: > >> Acho que

Re: [obm-l] Quadrados perfeitos

2019-11-27 Por tôpico Esdras Muniz
10^5([sqrt{12}]-1) Em qua, 27 de nov de 2019 08:57, marcone augusto araújo borges < marconeborge...@hotmail.com> escreveu: > Seja n E N tal que 1 < = n < = 10^10. Quantos números M = 11n + 10^10 > são quadrados perfeitos? > -- > Esta mensagem foi verificada pelo sistema de antivírus e > acredita

Re: [obm-l] Quadrados perfeitos

2019-11-27 Por tôpico Esdras Muniz
e 2019 às 13:41, Esdras Muniz < > esdrasmunizm...@gmail.com> escreveu: > >> 10^5([sqrt{12}]-1) >> >> Em qua, 27 de nov de 2019 08:57, marcone augusto araújo borges < >> marconeborge...@hotmail.com> escreveu: >> >>> Seja n E N tal que

Re: [obm-l] Quadrados perfeitos

2019-11-27 Por tôpico Esdras Muniz
Percebi agora que tô errado. Desculpa. Em qua, 27 de nov de 2019 19:22, Esdras Muniz escreveu: > Pensei assim, o 10^10= (10^5)^2 é qp, daí, (10^5+1)^2, (10^5+2)^2, ..., > [Sqrt{12×10^5}] são só quadrados que queremos contar. > > Estou usando [x] para demorar a parte interna de x. &g

Re: [obm-l] A c o i s a f i c o u f e i a e m 2 0 1 9

2019-11-28 Por tôpico Esdras Muniz
Primeiro, troque os hóspedes que já estão no hotel de quarto, mandando o hóspede do quarto n para o quarto 2n, assim, todos os quartos ímpares estarão desocupados. Depois, faça uma bijeção entre os ônibus e os naturais {1, 2, 3, ...}. Em seguida, faça uma bijeção entre os hóspedes do n-esimo ônibus

Re: [obm-l]

2019-11-28 Por tôpico Esdras Muniz
Acho que é (2019!)/(2^{1000}×1009!). Em qui, 28 de nov de 2019 12:41, Jamil Silva escreveu: > Qual o menor número que possui exatamente 2019 partições tal que em todas > elas as partes sejam números inteiros positivos e consecutivos ? > > -- > Esta mensagem foi verificada pelo sistema de antiví

Re: [obm-l]

2019-11-28 Por tôpico Esdras Muniz
, Jamil Silva escreveu: > Qual o raciocínio que leva a esse resultado ? > > > > Enviado do Email <https://go.microsoft.com/fwlink/?LinkId=550986> para > Windows 10 > > > -- > *De:* owner-ob...@mat.puc-rio.br em nome de > Esdras Mu

Re: [obm-l]

2019-11-29 Por tôpico Esdras Muniz
Tentei fazer o mesmo com R=1e l=√3, mas desisti qdo vi o tamanho das contas. Em sex, 29 de nov de 2019 16:09, Claudio Buffara escreveu: > Acho que com números complexos e alguma álgebra sai. > > Se os vértices do triângulo forem R, Rw e Rw^2 (onde w = cis(2pi/3) e R é > um real positivo) e P =

Re: [obm-l] Funcional equation

2019-12-09 Por tôpico Esdras Muniz
Vi um jeito de mostrar que só tem no máximo uma solução com grau n para cada n. Em ter, 10 de dez de 2019 00:11, Pedro Cardoso escreveu: > Minha intuição foi a seguinte, considere a sequência a_0=0 e > a_(n+1)=(a_n)²+1 > > Agora pomos P(0)=c > Pela equação funcional, P(0²+1)=c²+1 > E P((0²+1)²+1

Re: [obm-l]

2019-12-12 Por tôpico Esdras Muniz
Isso aí pode ir para o infinito: tome k real positivo arbitrário. Daí tome: (-k)+(-k)+...+(-k)+(n-1)k=0 (-k)^3+(-k)^3+...+(-k)^3+((n-1)k)^3=k^3((n-1)^3-(n-1)). Esse último fator vai pra o infinito com k. Em qui, 12 de dez de 2019 18:20, Anderson Torres < torres.anderson...@gmail.com> escreveu: >

[obm-l] Re: [obm-l] Teoria dos números

2019-12-13 Por tôpico Esdras Muniz
Existe congruência com números que não são inteiros? Em sex, 13 de dez de 2019 11:57, Prof. Douglas Oliveira < profdouglaso.del...@gmail.com> escreveu: > Olá caros amigos, > preciso de uma ajuda pra criar uma fórmula que seja congruente (módulo p) > ao somatório > S_a=sum{(a^k)/k}, com k de 1 a p

Re: [obm-l] soma com cevianas que passam pelo circuncentro

2019-12-19 Por tôpico Esdras Muniz
Eu tinha feito algo parecido com essa prova 2. Usando o método k. Em qui, 19 de dez de 2019 14:43, Luís Lopes escreveu: > Sauda,c~oes, > > Encontrei um link com a prova: > > https://www.cut-the-knot.org/m/Geometry/CeviansThroughCircumcenter.shtml > > Esse site é muito bom. > > Eu conhecia a prov

[obm-l] Re: [obm-l] Função Desconhecida

2019-12-20 Por tôpico Esdras Muniz
Acho que essa função é trancendente. Em sex, 20 de dez de 2019 14:42, Luiz Antonio Rodrigues < rodrigue...@gmail.com> escreveu: > Olá, pessoal! > Tudo bem? > Estou tentando, há alguns dias, resolver o seguinte problema: > > Preciso descobrir uma função f(x) cuja derivada é sen(x^3). Sabe-se que >

[obm-l] Re: [obm-l] Potência

2020-01-11 Por tôpico Esdras Muniz
Pode usar a função fi. Em sáb, 11 de jan de 2020 10:23, Vanderlei Nemitz escreveu: > Bom dia! > Eu resolvi essa questão, mas creio que trabalhei demais! > > Alguém conhece um modo relativamente simples? > > Os dois últimos algarismos de 2^222 são: > a) 84 > b) 24 > c) 64 > d) 04 > e) 44 > > Muit

[obm-l] Re: [obm-l] Potência

2020-01-11 Por tôpico Esdras Muniz
Acho que é d) 04 Em sáb, 11 de jan de 2020 11:01, Esdras Muniz escreveu: > Pode usar a função fi. > > Em sáb, 11 de jan de 2020 10:23, Vanderlei Nemitz > escreveu: > >> Bom dia! >> Eu resolvi essa questão, mas creio que trabalhei demais! >> >> Algué

Re: [obm-l] Soma de Riemann

2020-01-13 Por tôpico Esdras Muniz
Esse limite vai ser a integral inferior de sen(x) de 0 a b. Daí, como Sen é integravel, esse limite vai ser Sen(b). Em dom, 12 de jan de 2020 19:19, Luiz Antonio Rodrigues < rodrigue...@gmail.com> escreveu: > Olá, pessoal! > Tudo bem? > Estou pensando neste problema há vários dias e não consigo d

[obm-l] Re: [obm-l] Re: [obm-l] Re: [obm-l] Re: [obm-l] Re: [obm-l] Re: [obm-l] Questão OBM - U

2020-01-23 Por tôpico Esdras Muniz
É fácil ver que esse ínfimo tem que ser no mínimo 4, basta fazer desigualdade triângulos com os triângulos que têm dois vértices comuns com o quadrilátero e o terceiro sendo a interseção das diagonais. E por esse argumento do Caio, vemos que é 4 mesmo. Em qui, 23 de jan de 2020 08:59, Caio Costa

<    1   2